Difference between revisions of "2000 AMC 10 Problems/Problem 14"

(New page: 71, 76, 80, 82, 91. The sum of the first 2 must be even, so we must choose 2 evens or the 2 odds. Let us look at the numbers (mod 3). 2,1,2,1,1. If we choose the two odds, the next num...)
 
m
Line 1: Line 1:
 +
==Problem==
 +
 +
==Solution==
 +
 
71, 76, 80, 82, 91.
 
71, 76, 80, 82, 91.
  
Line 22: Line 26:
  
 
C.
 
C.
 +
 +
==See Also==
 +
 +
{{AMC10 box|year=2000|num-b=13|num-a=15}}

Revision as of 19:52, 8 January 2009

Problem

Solution

71, 76, 80, 82, 91.

The sum of the first 2 must be even, so we must choose 2 evens or the 2 odds.

Let us look at the numbers (mod 3).

2,1,2,1,1.

If we choose the two odds, the next number must be a multiple of 3, of which there is none.

Similarly, if we choose 76,80 or 80,82, the next number must be a multiple of 3, of which there is none.

So we choose 76,82 first.

The next number must be 1 (mod 3), of which only 91 remains.

The sum of these is 249. This is equal to 1 (mod 4).

Thus, we need to choose one number that is 3 (mod 4). 71 is the only one that works.

Thus, 80 is the last score entered.

C.

See Also

2000 AMC 10 (ProblemsAnswer KeyResources)
Preceded by
Problem 13
Followed by
Problem 15
1 2 3 4 5 6 7 8 9 10 11 12 13 14 15 16 17 18 19 20 21 22 23 24 25
All AMC 10 Problems and Solutions